LSAT and Law School Admissions Forum

Get expert LSAT preparation and law school admissions advice from PowerScore Test Preparation.

 Administrator
PowerScore Staff
  • PowerScore Staff
  • Posts: 8917
  • Joined: Feb 02, 2011
|
#22671
Question #3: Strengthen—PR. The correct answer choice is (C)

It takes a while to arrive at the author’s conclusion in question #3—not until the last sentence do we learn that the author believes some action would be unfair—so let’s first attempt to make sense of the situation described, and then consider the author’s take on it.

Louise McBride, in attempting to file a complaint against a nightclub, was given the wrong form by the agency in charge of these things (the Licensing Bureau), which she then used to file her complaint. However, because she used the wrong form the nightclub argues that the complaint should be dismissed.

The question then becomes one of fault rather than fact: should the complaint stand even though the wrong form was indeed used, since that error presumably lies with the Bureau and not the complainant?

The author clearly believes that the complaint should not be dismissed (dismissal would be “unfair”), as that would punish Ms. McBride for the Bureau’s mistake.

The correct answer choice will present a principle—a broad rule—that supports the author’s position that the complaint should stand.

Answer choice (A): does not provide any information about whether an improperly filed complaint should be dismissed if the filer is not at fault, so it cannot support the author’s position and is incorrect.

Answer choice (B): Like (A), this answer choice provides no context for interpreting the specific situation in the stimulus: nothing about governmental error or complaint dismissal is mentioned.

Answer choice (C): This is the correct answer choice. Note how closely this accords with the author’s conclusion—when there is an incorrect action by a government agency (Licensing Bureau’s assignment of the wrong form) it is “unfair for someone’s [Ms. McBride’s] complaint to be dismissed.” This is a perfect example of how Principle questions operate, where the generalizations in the principle will match neatly with the details of the specific scenario.

Answer choice (D): Again, nothing in this answer choice informs us that it would be unfair to dismiss a complaint based on the mistake of some governing body. Like both (A) and (B) we can dismiss (D) immediately.

Answer choice (E): Nothing in the stimulus discusses the prospect of the nightclub’s ability to defend itself against the complaint, so this answer too misses the mark. Granted, it is slightly better than (A), (B), and (D) in that it at least addresses the notion of “fairness,” but its introduction of the idea of defense still rules it out.

Get the most out of your LSAT Prep Plus subscription.

Analyze and track your performance with our Testing and Analytics Package.